When an observed difference between two groups on the dependent variable is attributed to the effect of the dependent variable through statistical hypothesis testing then we know that
it is unlikely that the difference would occur by chance alone

Answers

Answer 1

It is true that when difference on dependent variable is attributed to effect of that variable through statistical hypothesis testing then it is unlikely that the difference would occur by chance alone

A formal assertion about the nature of a phenomenon made within the constraints of a statistical model is known as a statistical hypothesis.

Statistical hypothesis is a description of the makeup of a population. A population parameter is frequently used to express it.

There are two statistical hypotheses used in hypothesis testing. The above-described null hypothesis (H0) is the first.

If it turns out that the null hypothesis is statistically invalid, an alternative hypothesis (Ha) for each H0 will be given preference. Depending on the research hypothesis, the Ha may be directed or nondirectional.

About the statement in the question,

It is improbable that an observed difference between two groups on the dependent variable would occur by chance alone when the difference is explained by the effect of the dependent variable through statistical hypothesis testing.

To know more about Statistical Hypothesis here

https://brainly.com/question/29112096

#SPJ4


Related Questions

Find X, 50 points if you answer

Answers

Answer:

x=38

Step-by-step explanation:

linear par

180-134=46

180-84=96

sum of a triangle is 180

96+46+x=180

142+x=180

x=180-142

x=38

Which polynomial represents the difference below?
8x³ + 5x+6-(2x² + 3x)
OA. 10x¹0 + 8x² +6
OB. -2x7 + 8x³ + 8x+6
O C. 6x¹0+2x+6
OD. -2x7 + 8x³ + 2x+6

Answers

The difference of the given polynomials 8x³+5x+6 and 2x²+3x is 8x³-2x²+2x+6. So, the correct answer is D.

What is the subtraction of polynomials?

To subtract polynomials from another, we should change the signs (from '+' to '-' or from '-' to '+') of all the terms of the expression which is to be subtracted and then the two expressions are added.

Given that, 8x³+5x+6-(2x²+3x)

Group the like terms are perform the addition or subtraction

= 8x³+5x+6-2x²-3x

= 8x³+(5x-3x)-2x²+6 (Here, like terms are 5x and 3x)

= 8x³+2x-2x²+6

= 8x³-2x²+2x+6

So, the standard form of obtained polynomial is 8x³-2x²+2x+6.

The polynomials difference is 8x³-2x²+2x+6. Therefore, option D is the correct answer.

To learn more about the subtraction of polynomials visit:

https://brainly.com/question/12959016.

#SPJ1

"Your question is incomplete, probably the complete question/missing part is:"

Which polynomial represents the difference below?

8x³ + 5x+6-(2x² + 3x)

A. 10x+ 8x² +6

B. -2x² + 8x³ + 8x+6

C. 6x¹⁰+2x+6

D. -2x²+ 8x³+2x+6

A manufacturer of banana chips would like to know whether its bag filling machine works correctly at the 414 gram setting. Based on a 8 bag sample where the mean is 407 grams and the standard deviation is 18, is there sufficient evidence at the 0.025 level that the bags are underfilled? Assume the population distribution is approximately normal.
Step 1 of 5:
State the null and alternative hypotheses.
Step 2 of 5:
Find the value of the test statistic. Round your answer to three decimal places.
Step 3 of 5:
Specify if the test is one-tailed or two-tailed.
Step 4 of 5:
Determine the decision rule for rejecting the null hypothesis. Round your answer to three decimal places.
Step 5 of 5:
Make the decision to reject or fail to reject the null hypothesis.
Question #2:
Our environment is very sensitive to the amount of ozone in the upper atmosphere. The level of ozone normally found is 4.8 parts/million (ppm). A researcher believes that the current ozone level is at an insufficient level. The mean of 26 samples is 4.6 ppm with a standard deviation of 1.2. Does the data support the claim at the 0.025 level? Assume the population distribution is approximately normal.
Step 1 of 5:
State the null and alternative hypotheses.
Step 2 of 5:
Find the value of the test statistic. Round your answer to three decimal places.
Step 3 of 5:
Specify if the test is one-tailed or two-tailed.
Step 4 of 5:
Determine the decision rule for rejecting the null hypothesis. Round your answer to three decimal places.
Step 5 of 5:
Make the decision to reject or fail to reject the null hypothesis.

Answers

A)

A manufacturer of banana chips would like to know whether its bag-filling machine works correctly at the 414-gram setting.

So, Null hypothesis: [tex]H_{0}[/tex] : μ < 414

It is believed that the machine is underfilling the bags.

So, Alternate hypothesis: [tex]H_{1}[/tex] : μ < 414

Given,

n= 8

Population standard deviation (б) = 18

x= 407

We will use the t-test since n > 8 and we are given the population standard deviation.

t=x-μ / (б/[tex]\sqrt{n-1}[/tex])

t= [tex]\frac{407-414}{\frac{18}{\sqrt{7} } }[/tex]

t= -1.028

Use the t table to find p value

p-value = 12.706

Level of significance α = 0.025

p-value>α

It is a two-tailed test.

So, we fail to reject the null hypothesis.

So, its bag-filling machine works correctly at the 414-gram setting.

B)

Let μ be the population mean amount of ozone in the upper atmosphere.

As per the given, we have

[tex]H_{0}[/tex]    : μ = 4.8

[tex]H_{1}[/tex] : μ ≠ 4.8

Sample size: n= 26

Sample mean = 4.6

Standard deviation = 1.2

Since population standard deviation is now given, so we use a t-test.

t= [tex]\frac{4.6-4.8}{\frac{1.2}{\sqrt{25} } }[/tex]

t= -0.2/0.24

t= -0.833

It is a two-tailed test.

We are accepting the null hypothesis.

To learn more about hypothesis testing visit: brainly.com/question/17099835

#SPJ4

Solve triangle ABC. (If an answer does not exist, enter DNE. Round your answers to one decimal place.) C 540 a 3.0, 4.0, LA = C = Solve triangle ABC. (If an answer does not exist,, enter DNE. Round your answers to one decimal place.) b 69 35, LA 72° C = C = a = Solve triangle ABC. (If an answer does not exist, enter DNE. Round your answers to one decimal place.) a 28, b = 39, c 29 LA = Solve triangle ABC. (If an answer does not exist, enter DNE. Round your answers to one decimal place.) = 17, 13, c 22 a = LA= o Sketch the triangle 500 LA B 770 C = 270 c 270 50° 77 50° 770 270 A A A 770 50° 270 270 50° 77 C A Solve the triangle using the Law of Sines. (Round side lengths to the nearest integer.) a = b Sketch the triangle. 100° LA = 270, C=60 C 100° 60 100° 27 27° C 60 C C 270 60 100° 27 100° A 60 A Solve the triangle using the Law of Sines. (Round side lengths to one decimal place.) a = b =

Answers

The measures of the lengths of the sides and angles of the triangles found using the law of cosines and the law of sines are presented as follows;

Question 1

∠A = 47.35°

∠B = 78.65°

c = 3.3

Question  2

∠B = 78.24°

∠C = 29.76°

a = 67.03

Question 3

∠A = 45.77°

∠B = 86.417°

∠C = 47.813°

Question 4

∠A = 36.15°

∠B = 60.48°

∠C = 93.37°

Question 5

a = 258.98

b = 327.41

∠C = 53°

Question 6

a = 34.11

b = 73.987

∠C = 53°

What is the law of cosines?

The law of cosines is a relationship between two sides (b and c) and the included angle, (∠A) and the third side (a) of the triangle.

Mathematically; a² = b² + c² - 2·b·c·cos(A)

Question 1

The dimensions of the triangle ΔABC are;

a = 3.0, b = 4.0, ∠C = 54°

The law of cosines indicates that we get;

c² = b² + a² - 2·b·a·cos(∠C)

Therefore;

c² = 3.0² + 4.0² - 2 × 3.0 × 4.0 × cos(54°) ≈ 10.893

c ≈ √(10.893) ≈ 3.3

The law of sines indicates that we get;

sin(54°)/3.3 = sin(∠A)/3.0

∠A = arcsine(3 × sin(54°)/3.3) ≈ 47.35°∠B = 180° - 54° - 47.35° ≈ 78.65°

Question 2

b = 69, c = 35, ∠A = 72°

a² = 69² + 35² - 2 × 69 × 35 × cos(72°) ≈ 4493.45

a ≈ √(4493.45) ≈ 67.03

The law of sines indicates that we get;

sin(72°)/67.03 = sin(∠B)/69

∠B = arcsine(69 × sin(72°)/67.03) ≈ 78·24°

∠B ≈ 78.24°

∠C = 180° - 72° - 78.24° ≈ 29.76°

∠C  ≈ 29.76°

Question 3

a = 28, b = 39, c = 29

a² = b² + c² - 2·b·c·cos(A)

cos(A) = (a² - (b² + c²)) ÷ (2·b·c)

Therefore; cos(A) = (28² - (39² + 29²)) ÷ (-2 × 39 × 29) ≈ 0.6976

∠A = arccos(0.6976) ≈ 45.77°

sin(45.77)/28 = sin(B)/39

sin(B) = 39 × sin(45.77)/28 ≈ 0.998

∠B = arcsine(0.998) ≈ 86.417°∠C = 180° - 45.77° - 86.417° = 47.813°

Question 4

a = 13, b = 17, c = 22

cos(A) = (13² - (17² + 22²)) ÷ (-2 × 17 × 22) ≈ 0.807

∠A ≈ arccos(0.807) ≈ 36.15°

sin(36.15)°/13 = sin(∠B)/17

sin(∠B) = 17 × sin(36.15)°/13

∠B =50.48°           ∠C = 180° - 36.15° - 50.48° ≈ 93.37°

Question 5

The parameters of the triangle are; ∠A = 50°, ∠B = 77°, c = 270

Please find attached the sketch of the triangle in the correct option created with MS Word

∠C = 180° - 50° - 77° = 53°

a/sin(50°) = 270/sin(53°)

a = sin(50°) × 270/sin(53°) ≈ 258.98

a = 258.98

b = sin(77°) × 270/sin(53°) ≈ 329.41

b ≈ 329.41

Question 6

The parameters of the triangle are;

∠A = 27°, ∠B = 100°, c = 60

Please find attached the drawing of the correct triangle

∠C = 180° - 27° - 100° = 53°

∠C = 53°

60/sin(53°) = a/sin(27°)

a = sin(27°) × 60/sin(53°) ≈ 34.11

b = sin(100°) × 60/sin(53°) ≈ 73.987

Learn more on the law of sines here: https://brainly.com/question/14437641

#SPJ1

Select all of the lines of reflection that will carry the rectangle back onto itself.

Answers

The lines that carry the rectangle onto itself are x = 0 and y = 1

How to determine the lines that carry the rectangle onto itself?

The graph that completes the question is added as an attachment

From the question, we have the following parameters that can be used in our computation:

The rectangular graph

The coordinates of one end of the graph are

(-3, 3) and (-3, -1)

Next, we calculate the midpoint of these ends

So, we have

Midpoint = 1/2(x₁ + x₂, y₁ + y₂)

Substitute the known values in the above equation, so, we have the following representation

Midpoint = 1/2(-3 + 3, -1 + 3)

Evaluate the like terms

Midpoint = 1/2(0, 2)

So, we have

Midpoint = (0, 1)

So, we have

x = 0 and y = 1

Hence, the reflection lines are x = 0 and y = 1

Read more about reflection at

https://brainly.com/question/27224272

#SPJ1

The volume of a rectangular prism is 6,618.375 cm3. If the height is 13.25 cm and the length is 27 cm, what is the value of the width?

A: 18.125 cm
B: 18.5 cm
C: 18.75 cm
D: 18.86 cm

Answers

The value of width will be;

⇒ 18.5 cm

What is an expression?

Mathematical expression is defined as the collection of the numbers variables and functions by using operations like addition, subtraction, multiplication, and division.

Given that;

The volume of a rectangular prism = 6,618.375 cm³

The height is 13.25 cm and the length is 27 cm.

Now,

We know that,

The volume of rectangular prism = Length x Width x Height

Substitute all the values, we get;

⇒ 6,618.375 = 27 × x × 13.25

⇒ 6,618.375 / 357.75 = x

⇒ x = 18.5 cm

Thus, The value of width = 18.5 cm

Learn more about the mathematical expression visit:

brainly.com/question/1859113

#SPJ1

One hundred elk, each 1 year old, are introduced into a game preserve. The number N(t) alive after t years is predicted to be N(t)=100(0.9)^t
(a) Estimate the number alive after 7 years. (Round your answer to the nearest whole number.)
(b) What percentage of the herd dies each year?

Answers

a) The number alive after 7 years is given as follows: 48.

b) The percentage of herd that dies each year is of 10%.

What is the exponential function?

The exponential function in the context of this problem is defined as follows:

N(t)=100(0.9)^t.


The parameters of the function are defined as follows:

y-intercept of 100, which is the number of elk alive at year 0.Decay rate of 0.1 = 10%, as 1 - r = 0.9, meaning that the percentage of the herd that dies each year is of 10%.

The amount of herd alive after 7 years is found with the numeric value at t = 7, replacing the lone instance of t in the function by 7, hence:

N(7) = 100 x (0.9)^7 = 48.

(rounding to the nearest whole number).

More can be learned about exponential functions at https://brainly.com/question/25537936

#SPJ1

A side of the triangle below has been extended to form an exterior angle of
132°. Find the value of x

Answers

Answer:

x = 15°

Step-by-step explanation:

the exterior angle of a triangle is equal to the sum of the angles that is opposite of said exterior angle

Thus, to solve for x, we have to make an equation in regards of the exterior angle.

132° = x° + 117°

∴ 15 = x° . . . .  subtract 117 from LHS and RHS

⭐if this response helped you, please mark it the "brainliest"!⭐

t²-8t+16 can be factorized to give an expression of the form (t + a)², where a is an integer.
Work out the value of a.

Answers

Answer:

a = 4

Step-by-step explanation:

Step 1: Find two numbers that multiply to give 16 and add to give -8.

t²-8t+16 = 0

t² -4t -4t + 16 = 0

The two numbers are 4 and -4.

Step 2: Rewrite the equation in the form (t + 4)(t - 4).

t² - 8t + 16 = (t + 4)(t - 4)

Step 3: Factor the equation to get (t + 4)².

(t + 4)² = (t + 4)(t + 4)

Therefore, a = 4.

To learn more about factorization refer to:

https://brainly.com/question/25829061

#SPJ1

Sydney went to the store and bought candy that was priced according to the weight in pounds. She purchased 2 1/4 pounds of black licorice, 1 7/8 pounds of red licorice, and 1 1/2 pounds of butterscotch candy. if the candy costs $ 4.00 per pound, how much did Sydney spend on candy?

Answers

Answer:

$22.50

Step-by-step explanation:

NO LINKS!! Please help me with this problem. Part 8ff​

Answers

Answer:

[tex]\dfrac{1}{36n^2+6n}[/tex]

Step-by-step explanation:

Given factorial expression:

[tex]\dfrac{(6n-1)!}{(6n+1)!}[/tex]

[tex]\boxed{\begin{minipage}{6cm}\underline{Factorial Rule}\\\\$n!=\:n\cdot \left(n-1\right) \cdot \left(n-2\right) \cdot ... \cdot 3 \cdot 2\cdot 1$\\ \end{minipage}}[/tex]

Apply the factorial rule to the numerator and denominator of the given rational factorial expression:

[tex](6n-1)!=\left(6n-1\right)\cdot \left(6n-2\right)\cdot \left(6n-3\right)\cdot... \cdot 3 \cdot 2\cdot 1[/tex]

[tex]\left(6n+1\right)!=\left(6n+1\right)\cdot \:6n \cdot (6n-1) \cdot...\cdot 3 \cdot 2\cdot 1[/tex]

Therefore:

[tex]\begin{aligned}\implies \dfrac{(6n-1)!}{(6n+1)!}&=\dfrac{\left(6n-1\right)\cdot \left(6n-2\right)\cdot \left(6n-3\right)\cdot... \cdot 3 \cdot 2\cdot 1}{\left(6n+1\right)\cdot \:6n \cdot (6n-1) \cdot...\cdot 3 \cdot 2\cdot 1}\\\\&=\dfrac{1}{(6n+1) \cdot 6n}\\\\&=\dfrac{1}{6n(6n+1)}\\\\&=\dfrac{1}{36n^2+6n}\end{aligned}[/tex]

Answer:

[tex]\cfrac{1}{6n(6n+1)}[/tex]

--------------------------------

We know that:

n! = 1·2·3·4·...·n

Therefore:

(6n + 1)! = (6n - 1)!·6n·(6n + 1)

Therefore:

[tex]\cfrac{(6n-1)!}{(6n+1)!} =\cfrac{(6n-1)!}{(6n-1)!(6n)(6n+1)} =\cfrac{1}{6n(6n+1)}[/tex]

The marketing department of a large chain of automobile tire retail stores would like to pursue consumers interested in all-terrain tires. They wish to investigate the extent to which the amount of money spent on TV advertising (ADV) on Sundays is related to the sales revenue (REV) for the week- from this type of tire. They begin by selecting a random sample of 50 stores in various cities. The analyst looks at the data and notices that 9 of the stores are located in cities with warm climater year-round, and there are no off-road driving possibilities anywhere near these locations. She suggests eliminating these cities from the sample as she feels that money spent advertising these tires will have little, if any, effect on sales. Using the remaining cities, a simple regression model is determined. Regression Analysis 7² 0.862 #41 > 0.929 5, 915.247 Dep. Var. REV Regression output variables std. error Intercept

Answers

If a store decides to spend $1500 on Sunday TV ads for all-terrain tires, we can predict that 95% of the values for the revenue in those weeks from the sale of these will be between $21863.59 and $25617.41.

A company can use regression analysis to determine statistical relationships between the total sales of its products and the costs associated with promotions and advertising. The outcomes of these analyses can then be used to determine how best to allocate spending funds among, say, various products or sales regions.

n = 41

df = n - 2

df = 41 - 2 = 39

SSₓₓ = (n - 1)S²ₓ

SSₓₓ = (41 - 1) * (400)²

SSₓₓ = 6400000

y = 15531.3086 + 5.4728 * x

If, x = 1500 then:

y = 15531.3086 + 5.4728 * 1500

y = 23740.51

Prediction range of y at 95% = [tex]y + t_{\frac{0.05}{2},39 } * S_{e} * \sqrt{1 + \frac{1}{n} + \frac{(x - x_{1} )^{2}}{SS_{xx} } }[/tex]

Prediction range of y at 95% = [tex]23740.51 + 2.022691 * 915.247 * \sqrt{1 + \frac{1}{41} + \frac{(1500 - 1700)^{2} }{6400000}[/tex]

Prediction range of y at 95% = (2186.1 , 25619.92)

To know more about Regression Analysis, refer to this link:

https://brainly.com/question/28298210


#SPJ4

Find the volume of a cone with a radius of 3 feet and a height of 7 feet. Enter
the answer in terms of pie

Answers

Answer: volume of cone = 21 π ft^3

What is the rate of return when 12 shares of Stock
A, purchased for $22/share, are sold for $465? The
commission on the sale is $9.
Rate of Return
Enter the appropriate value into the
formula to calculate the rate of return.
F
profit or loss
total cost
Total Cost = $273
Profit = $192
Rate of Return = [? ]

Answers

Answer:

The Rate of return would then be 192 / 273 ≈ 70.32%

Can anyone solve I need help urgent thank you

Answers

Answer:

Step-by-step explanation:

3.14 x 3=9.42

10 -8 -6 -4
| 10+
8
67
-2
4.2
-2
-4-
-6
-8
-10
2
4 6 8 10
Write an equation for the graph, where y depends on x.

Answers

The equation of given graph is y = 2x + 6.

What is equation of line?

The formula for a straight line is y = mx + c where c is the height at which the line intersects the y-axis, also known as the y-intercept, and m is the gradient.

Given:

The graph of the line is given.

From graph we have to find the equation of line.

Let the graph passes through the points (0, 6) and (2, 10).

From these two points to find the slope.

Slope = [tex]\frac{y_2 - y_1}{x_2 - x_1}[/tex]

Here, [tex](x_1, y_1) = (0, 6), (x_2, y_2) = (2, 10)[/tex]

⇒ Slope = m = [tex]\frac{10-6}{2-0}= \frac{4}{2} = 2[/tex]

So, the slope is 2.

Now to find the equation of line.

Consider, the point - slope form of the line,

[tex]y-y_1=m(x-x_1)[/tex]

Plug [tex]m = 2, (x_1, y_1) = (0, 6)[/tex]

[tex]y-6=2(x-0)\\y-6=2x\\y=2x+6[/tex]

Hence, the equation of given graph is y = 2x + 6.

To learn more about equation of line, click on the link

https://brainly.com/question/18831322

#SPJ1

A function is shown in the table below:

What is the average rate of change of the function from X equals -7 to X equals 2?

A. -90/101

B. 101/90

C. -101/90

D. 90/101

Answers

The average rate of change of the given function from x equals -7 to x equals 2 is calculated as: B. 101/90.

How to Find the Average Rate of a Function for a Given Interval?

If we are given a function, f(x), to find the average rate of change of the function within the interval from a to b, the formula to use is:

Average rate of change = f(b) - f(a) / b - a.

Given the table that represents  function, we have the following:

a = -7

b = 2

f(a) = f(-7) = -2.7

f(b) = f(2) = 7.4

Plug in the values into the formula:

Average rate of change = (7.4 - (-2.7)) / (2 - (-7))

= (7.4 + 2.7) / 2 + 7)

= 10.1 / 9

= 101/90

Therefore, the average rate of change is: B. 101/90.

Learn more about average rate of change of a function on:

https://brainly.com/question/26096632

#SPJ1

Suppose that the cost C (in dollars) of removing p percent of the particulate pollution from the smokestacks of an industrial plant is given by
C(p) =
8100p
100 − p
(a) Is C(p) undefined at any p-value? If so, what value? (If an answer does not exist, enter DNE.)
p =
(b) What is the domain of C(p) as given by the equation? (Enter your answer using interval notation.)
(c) What is the domain of C(p) in the context of the application? (Enter your answer using interval notation.)
(d) What happens to the cost as the percent of pollution removed approaches 100%?
The cost increases (bounded by 8100) as p increases.
The cost decreases (bounded by 0) as p increases.
The cost decreases without bound as p increases.
The cost increases without bound as p increases.
The cost remains constant a

Answers

(a) C(p) is not defined since we can't divide by 0.

(b) The domain of C(p) as given by the equation is (-∞, 100) ∪ (100, ∞)

(c) The domain of C(p) in the context of the application is [0, 100)

(d) The percentage of particulate pollution removed from an industrial plant can't be negative or higher or equal than 100

Here we have given that Suppose that the cost C (in dollars) of removing p percent of the particulate pollution from the smokestacks of an industrial plant is given by

C(p) = 8100p / (100 − p)

And we need to find the following.

(a) In general ma the we know that dividing by zero is not possible one.

Therefore, C(p) is undefined when we try to divide it by 0.

(b) The domain of C(p) is the function is not defined at p =100 the domain would be written as,

=> D = (-∞, 100) ∪ (100, ∞)

(c) The domain of C(p) in the context of the application  is calculated as,

=> D = [0, 100)

Therefore, the percentage of particulate pollution removed from an industrial plant can't be negative or higher or equal than 100.

To know more about Domain of the function here.

https://brainly.com/question/13113489

#SPJ4

find the probability of rolling a five or six on six sided number cube

Answers

Answer:

non-violence and kindness with animals

Step-by-step explanation:

-x+y≤-1
x + 2y ≥ 4
Graph the system of inequalities.

Answers

Answer:

Step-by-step explanation:

[tex]-x+y\leq -1\\x-y\geq 1\\x+2y\geq 4[/tex]

dark blue is the required region.

and drop each item into the correct column. Order does not matter:

Answers

(1, 5); (4, 7) - corresponding, (3, 5) - Alternate Interior, (2, 7); (1, 8) - Exterior

(3, 6) are Consecutive Interior Angles.

Line of transversal:

Any line that crosses two straight lines at different locations is known as a transversal.

Corresponding Angles:

When two parallel lines cross the third one, the angles that are in the same relative position at each junction are referred to as corresponding angles to one another

Alternate Interior Angles:

The alternative interior angles are those that are on the inner side of the parallel lines but on the opposing sides of the transversal.

Alternate Exterior Angles:

Alternate external angles are positioned on the opposing sides of the transversal and are always outside the two lines where the transversal intersects.

Consecutive Interior Angles:

The pair of non-adjacent internal angles that are located on the same side of the transversal are referred to as consecutive interior angles.

Here we have

Two parallel lines intersected and formed the angles, ∠1, ∠2, ∠3, ∠4, ∠5, ∠6, ∠7, and ∠8.

By the given information,

∠1  and ∠ 5 are corresponding angles

∠4 and ∠ 7 are corresponding angles

∠3 and ∠5 are Alternate Interior Angles

∠2, and ∠7 are Alternate Exterior angles

∠1 and ∠ 8 are Alternative Exterior angles

∠3 and ∠ 6 are Consecutive Interior Angles

Therefore,

(1, 5); (4, 7) - corresponding, (3, 5) - Alternate Interior, (2, 7); (1, 8) - Exterior

(3, 6) are Consecutive Interior Angles.

Learn more about the Line of transversal at  

https://brainly.com/question/9610896

#SPJ1

Solve for x. Triangle stuff

Answers

Answer:

x=9

Step-by-step explanation:

these 2 angles are supplementary angles meaning added together they will equal 180 degrees

so we can add them together and set it equal to 180

(8x-3)+(16x-33)=180

combine like terms

(8x+16x)+(-3-33)=180

24x-36=180

     +36. +36

24x=216

/24.  /24

x=9

hopes this helps

1 hour 15 minutes. is what in minutes

Answers

Answer:

75

Step-by-step explanation:

I hour and 15 minutes in minutes is 75 minutes

1 hour = 60 minutes

you add 15 and you get

60+15=75

The answer would be 75 Minutes.

1 Hour = 60 Minutes

[tex]60 + 15 = 75[/tex] Minutes.

(a) You have a 10 inch by 15 inch piece of tin which you plan to form into a box (without a top) by cutting a square from each corner and folding up the sides. How much should you cut from each corner so the resulting box has the greatest volume? (b) If the piece of tin is A inches by B inches, how much should you cut from each corner so the resulting box has the greatest volume?

Answers

Resulting box has the greatest volume for the values  (25 ± 5√7)/6 .

This is a problem that can be solved using derivatives , maxima & minima and common logic.

Hence , going by logic :

Creating a flap of 'a' inches in width, the base of the box will be

 (10 - 2a) by (15 - 2a)

and the depth of the box will be the width of the fold-up flap: a.

Then the volume of the box is

 v = [tex]a(10 -2a)(15 -2a) = 150a -50a^2 +4a^3[/tex]

Using the derivative of the volume will be zero at the maximum volume.

 0 = [tex]dv/da = 150 -100a +12a^2[/tex]

This has roots at

 a = (100 ±√(100² - 4(12)(150)))/(2·12)

 a = (100 ± √2800)/24 = (25 ± 5√7)/6

Only the smaller of these solutions gives a maximum volume.

You should cut (5/6)(5-√7) ≈ 1.962 inches to obtain the greatest volume.

Similarly , replacing the values of 10 by A and 15 by B , a generalized solution can be formed .

To know more about maxima and minima, go to brainly.com/question/29562544

#SPJ4

Rosa makes candles to sell.
Each candle is in the shape of a cuboid of height 8 cm.
The base of each candle is a square of perimeter 20 cm.
Rosa needs to know the volume of one candle.
Work out the volume of one candle.
Remember to give units with your answer

Answers

To find the volume of the candle, you need to find the volume of the cuboid. The volume of a cuboid is found by multiplying the length, width, and height.

First, you need to find the length and width of the base. The perimeter of the base is 20 cm, and since the base is a square, all four sides are the same length. You can divide the perimeter by 4 to find the length of each side:

20 cm / 4 = 5 cm

So, each side of the square base has a length of 5 cm. This means the length and width of the base are both 5 cm.

Now that you know the length, width, and height of the cuboid, you can calculate the volume:

Volume = length * width * height
= 5 cm * 5 cm * 8 cm
= 200 cm^3

So, the volume of the candle is 200 cm^3.

use the mean value theorem to verify that at some time during the first 3 seconds of fall the instataneous velocity equals the average velocity

Answers

Verified that during the first 3 seconds of fall the instantaneous velocity equals the average velocity, the time is 1.5 seconds

The height of an object t seconds after it is dropped from a height of 300 meters is

s(t) = -4.9t^2 + 300

The average velocity of object during first 3 seconds

= s(3) - s(0) / 3 - 0

s(3) = -4.9 × (3)^2 + 300

= -44.1 + 300

= 255.9 meters

s(0) = -4.9(0)^2 + 300

s(0) = 300

The average velocity = (255.9 - 300) / 3 - 0

= -44.1 / 3

= -14.7 meter per second

To find  instantaneous velocity differentiate the function

= -9.8t

The instantaneous velocity = The average velocity

-9.8t = -14.7

t = -14.7/-9.8

t = 1.5 seconds

Therefore, time is 1.5 seconds

I have answered the question in general, as the given question is incomplete

The complete question is :

The height of an object t seconds after it is dropped from a height of 300 meters is s(t) = -4.9t^2 + 300.

(b) Use the Mean Value Theorem to verify that at some time during the first 3 seconds of fall, the instantaneous velocity equals the average velocity. Find that time.

Learn more about average velocity here

brainly.com/question/27982972

#SPJ4

1. Using the stopping distance calculator and your internet browser complete the following table to compare stopping distances at various speeds.

2. You overhear your friend say “It is OK to go 10 mph over the speed limit.” Explain why your friend is wrong using your evidence from the table above to support your answer.

Answers

The stopping distance table and the analysis of the stopping distance are presented as follows;

1. The stopping distance calculator, an online tool, provides the stopping distance at a specified speed as presented in the table on the following sections.

2. Increasing the vehicle speed by 10 mph, increases the required stopping distance exponentially, which reduces the safety of driving

What is the stopping distance?

The stopping distance is the distance traveled by a vehicle, which is the sum of the distance traveled during the reaction time and the distance traveled during braking (the braking distance)


The table in the question using a perceptionreaction time of 2.5 seconds, and the online stopping distance calculator is completed as follows;

Speed (mph)       [tex]{}[/tex] Speed (m/s)        Stopping Distance (m)

60 mph [tex]{}[/tex]                 26.82 m/s           119.55 m

50 mph  [tex]{}[/tex]                22.35 m/s           92.34 m

40 mph  [tex]{}[/tex]                17.88 m/s            68.05 m

30 mph   [tex]{}[/tex]               13.41 m/s             46.665 m

20 mph [tex]{}[/tex]                 8.94 m/s             28.197 m

10 mph [tex]{}[/tex]                  4.47 m/s             12.642 m

2. The details from the above table indicates that as the speed increases, the stopping distance increases exponentially, such that increasing the speed by 10 mph increases the required stopping distance when the vehicle is moving at an already high speed, thereby reducing safety by increasing the speed by 10 mph.

Learn more about stopping distance here:

https://brainly.com/question/5254029

#SPJ1

let $f(x)$ be a polynomial with integer coefficients. suppose there are four distinct integers $p,q,r,s$ such that $$f(p)

Answers

The smallest possible value of f ( t ) = 9 based on the values of p , q , r , s.

Given :

Let f ( x ) be a polynomial with integer coefficients. Suppose there are four distinct integers p , q , r , s such that f ( p ) = f ( q ) = f ( r ) =f ( s ) = 5. If t is an integer and f ( t ) > 5,

Let g(x) = f(x) − 5.

g(x) = (x−p)(x−q)(x−r)(x−s)h(x)

The condition f(t) > 5 translates to g(t) > 0.

Since p,q,r,s,t are distinct integers, the smallest possible positive value of (t−p)(t−q)(t−r)(t−s) is 4 :

the four numbers in the parentheses are all distinct integers ≠ 0, so the smallest value we can get from the product (−2)⋅(−1)⋅1⋅2. }

The smallest possible positive value of h(t) is 1, since we must have g(t)≠0.

Thus the smallest possible value of g(t) is 4, and therefore the smallest possible value of f(t) is 9, and it is achieved for t=2 if we have

f(x)=x(x−1)(x−3)(x−4)+5

Learn more about the polynomial here:

https://brainly.com/question/11536910

#SPJ4

Full question ;

Let f(x) be a polynomial with integer coefficients. Suppose there are four distinct integers p,q,r,s such that f(p)=f(q)=f(r)=f(s)=5. If t is an integer and f(t)>5, what is the smallest possible value of f(t)?

A customer is buying bath towels and hand towels and can spend no more than $100. Each bath towel costs $8, and each hand towel costs $5. The inequality 8x+5y ≤100 represents all possible combinations of x, the number of bath towels, and y, the number of hand towels the customer can buy.
Which graph best represents the solution set for this inequality?

Answers

There is an attachment that includes the graph for the inequality 8x + 5y 100.

How do you calculate the scenario's graph?

The parameters are as follows, taken from the query:

The customer's purchase must not exceed $100.

Each bath towel costs $8.

Each hand towel costs $5.

Inequality among the available combinations is another factor that we have.

8x + 5y ≤ 100

like that

x is the quantity of bath towels, while y is the quantity of hand towels.

Plotting the inequality's graph is the following step.

A graphing tool can be used for this.

In order to attach the graph, we enter the inequality in the graphing tool.

Keep in mind that the inequality is provided as

8x + 5y ≤ 100

See the inequality graph in the attached. 8x + 5y ≤ 100

Read more about inequality here.

brainly.com/question/25275758

#SPJ1

What is it? I swear I have no idea

Answers

By using matrix, it can be calculated that:

[tex]\frac{1}{4}C = \begin{bmatrix}3 & 4 & -5 \\ 1 & -6 & 7\end{bmatrix}[/tex]

What is a matrix?

The term "matrix" refers to any configuration of numbers in the form of rows and columns. a collection of numbers lined up in rows and columns to form a rectangular array is called a matrix. The elements, or entries, of the matrix are the integers.

In addition to numerous mathematical disciplines, matrices find extensive use in the fields of engineering, physics, economics, and statistics. Solving linear equations is made easier by it. Matrices are incredibly priceless items that are used in a variety of contexts. In addition to mathematical applications, matrices are employed in a wide range of scientific disciplines. Nearly every element of our life uses engineering mathematics.

Here,

[tex]C = \begin{bmatrix} 12 & 16 & -20 \\ 4 & -24 & 28 \end{bmatrix}[/tex]

[tex]\frac{1}{4}C = \begin{bmatrix} 12\times \frac{1}{4} & 16 \times \frac{1}{4} & -20 \times \frac{1}{4} \\ 4 \times \frac{1}{4} & -24 \times \frac{1}{4}& 28 \times \frac{1}{4}\end{bmatrix}\\\\\frac{1}{4}C = \begin{bmatrix}3 & 4 & -5 \\ 1 & -6 & 7\end{bmatrix}[/tex]

To learn more about matrix, refer to the link-

https://brainly.com/question/94574

#SPJ1

Other Questions
The table and grab below shows the distance travel between two different objects which statement is correct about the speed of the two objects When the pCO2 of the blood increases, the production of H+ ___ but only ___ is capable of crossing the blood-brain barrier to elicit a response by central chemoreceptors. Which of the following sentences is written CORRECTLY?Students learn the value of timeliness and the effort that is required to fulfill a commitment.Students learn the value of timeliness, and the effort that is required to fulfill a commitment.Students learn the value of timeliness. And the effort that is required to fulfill a commitment.Students learn the value of timeliness and the effort. That is required to fulfill a commitment.Leave it as is. When the market rate is 10%, a company issues $60,000 of 12%, 10-year bonds dated January 1, 2017, that mature on December 31, 2026, and pay interest semiannually. When the bonds mature, the issuer records its payment of principal with a (debit/credit) Credit to Cash in the amount of $5000. based on the graphic, in what part of the electromagnetic spectrum does vegetation have the strongest response? a student who specializes in neurology in their doctorate of physical therapy program would be best equipped to work with which patient conditions? (choose all that apply) What was one effect of this new style of "musical western" on the united states in the 1930s?. ABCD with vertices B (2,1), C (2,5), and D (6,1) has its orthocenter at which point?A. (0,0)B. (2,1)C. (2,5)D. (6,1) ___ is a system for classifying people based on perceived innate physical characteristics, whereas _____ is a system for classifying people who are percieved to share cultural similaritiesrace; ethnicity Help me !!??!!!????please under which of the following does a minor enter into a binding contract: a) infancy doctrine; b) disaffirmance; c) necessaries of life; d) none of the above. a post-menopausal woman with osteoarthritis of the left hip underwent a total hip replacement. after the surgery, the patient suffered total paralysis of the left quadriceps muscle group and radiating pain along the posterior thigh. it was determined that nerves from the lumbar plexus and the sacral plexus were damaged during the surgery. what two nerves were most likely damaged? What is the main benefit of an upwelling in the ocean?OA.OB.O C.OD.It creates strong tides in coastal regions.It increases the salinity of the water in an area.It brings new nutrients into an underwater region.It wipes out an ecosystem, allowing for secondary succession. Which of the following factors should managers use in determing in-group/out-group membership in reciprocal theory? Multiple Choice similarity between the employee and the manager blases of the manager performance of the employee likeablity of the employee all of the above should be used by a manager in determing in-group/out-group membership for years, some foreign firms hesitated to send women managers on foreign assignments to countries, such as , because the firms assumed the women would not be accepted in the culture that frowned on women working outside the home. a. canada b. england c. australia d. japan The proportion of households that own mutual funds but not individual stocks or individual stocks but not mutual funds is a. 40%. b. 60%. c. 80%. d. 100% A triangle has vertices (0,0), (2,4), and (5,1). What are the coordinates of the circumcenter?I need steps pls Mr. Frankel bought 5 tickets to a puppet show and spent $28. He bought a combination of child tickets for $2 each and adult tickets for $8 each. Which system of equations below will determine the number of adult tickets, a, and the number of child tickets, c, he bought?PLEASE HELP ASAP ASAP ASAP Adriana has been experiencing a great deal of pain for the last three years as she copies with breast cancer. This type of pain is classified as ___ a data analyst uses the annotate() function to create a text label for a plot. which attributes of the text can the analyst change by adding code to the argument of the annotate() function? select all that apply.